Round the following numbers to the nearest 10. 13-21-29

Round The Following Numbers To The Nearest 10. 13-21-29

Answers

Answer 1

Answer:

13 --> 10

21 --> 20

29 --> 30

Answer 2

Answer:

The correct answer is: 10-20-30

Step-by-step explanation:

13 is between 10 and 20. The closest 10 for 13 is 10, because it's only 3 away (13  the other option would be 20 but it's 7 away, so way more.)

21 is between 20 and 30. The closest 10 for 21 is 20 ( it's only 1 away from 20, the other option would be 30 but it's 9 away, so way more.)

29 is between 20 and 30. The closest 10 for 29 is 30 (it's only 1 away from 30, the other option would be 20 but it's 9 away, so way more.)


Related Questions

solve

-3(x+5)=-9
Enter your answer in the box.

x=​

Answers

Answer:

x=-2

Step-by-step explanation:

-3(x+5)=-9

x+5 = 3

x=-2

What is the solution of 12b=18

Answers

Answer:

b=18/12

b=3/2

so practice more

if one gallon of paint covers 825 square feet, how much paint is needed to cover 2640 square feet? Make a prediction ​

Answers

Answer:

3.2 gallons

Step-by-step explanation:

Write a proportion:

1 gallon / 825 square feet = x / 2640 square feet

Solve:

x = 3.2 gallons

This year, a small business had a total revenue of $45,000. If this is 25% less than their total revenue the previous year, what was their total revenue the previous year?

Answers

Let R = total revenue the year before

R = 45,000 x 0.25 + 45,000

R = 11,250 + 45,000

R = $56,250

Find the solutions to x2 = 20.
:):):) i love you

Answers

Answer:

x = 10

Step-by-step explanation:

1st Divide both sides by 2: x2 ÷ 2 = 20 ÷ 2

2nd Simplify: x = 10

Hope I helped :)

Final answer:

The equation x² = 20 has two solutions: x = 2√5 and x = -2√5, as both positive and negative square roots are considered for the value of x.

Explanation:

To find the solutions for the equation x² = 20, we need to take the square root of both sides of the equation.

Generally, the square root of a variable squared, such as x², can be either positive or negative. This is because both a positive and a negative number, when squared, give the same positive result. Therefore, the equation has two solutions:

x = √20

x = -√20

Using a calculator or knowing that √4 ×√5 = 2 ×√5, we can simplify √20 to 2√5.

The final solutions are:

x = 2√5

x = -2√5

Which two whole numbers lies between the square root of 210?

Answers

Answer: 6√35

Step-by-step explanation:

since the √ 210 cannot come out into an equal number then you have to try to get two numbers closest to answering 210 to get it except you have to do 6 square root of 35. I hope this helps you!!!

Final answer:

The two whole numbers between which the square root of 210 lie are 14 and 15.

Explanation:

The square root of 210 is approximately 14.49. Since you are looking for the two whole numbers between which this value lies, you need to find the next higher and lower whole numbers from this value. These would be 14 and 15, as 14.49 is more than 14 and less than 15.

Learn more about Square Root here:

https://brainly.com/question/1540542

#SPJ2

Convert the following Celsius degrees to Fahrenheit 35° F

Answers

Answer:

Step-by-step explanation:

95 degrees farighite

Find the domain of the function.
g(x) = 1 - 2x^2

Answers

The domain of the function will be Domain: ( − ∞ , ∞ ) , { x | x  ∈  R }.

What is the domain of the function?

The entire range of independent variable values is the domain of a function. In algebra, a polynomial function with one or more variables in which the highest-degree term is of the second degree is known as a quadratic function, a quadratic polynomial, a polynomial of degree 2, or simply a quadratic.

The given quadratic function g(x) = 1 - 2x² will have a domain from negative infinity to positive infinity. The graph of the function is also attached with the answer below.

To know more about the domain follow

https://brainly.com/question/26098895

#SPJ2

The width of a rectangle is five less than three times the length of the worktable. If the perimeter of the rectangle is 70 inches what are the dimensions of the rectangle?

Answers

Answer:

The dimensions of rectangle whose perimeter is 70 inches and width is five less than three times the length of worktable is  length = 10 and width = 25

Solution:

Consider ‘P’ as perimeter and ‘L’ as length and ‘W’ as width  

P = 70 inches

W = 5 less than thrice of length  

Therefore, [tex]\mathrm{W}=3 L-5 \quad(\text {equation } 1)[/tex]

Using perimeter formula for rectangle,

[tex]\begin{array}{l}{P=2(L+W)} \\ {70=2(L+3 L-5)} \\ {70=2(4 L-5)} \\ {70=8 L-10} \\ {8 L=80} \\ {L=10}\end{array}[/tex]

Substitute L= 10 in equation 1

[tex]\begin{aligned} W &=3 \times 10-5 \\ W &=30-5 \\ W &=25 \end{aligned}[/tex]

simplify 3 to the 2nd power times 3 to the third power. ​

Answers

[tex]\bf 3^2\cdot 3^3\implies 3^{2+3}\implies 3^5\implies 243[/tex]

Final answer:

To simplify 3 to the 2nd power times 3 to the third power, we add the exponents because the base is the same, resulting in 3 to the 5th power, which equals 243.

Explanation:

To simplify 3 to the 2nd power times 3 to the third power, we need to apply the rule of multiplying exponential terms with the same base. When multiplying terms like 32 and 33, we simply add the exponents together because the base (the number 3 in this case) remains the same. So this simplification process looks like:

32 × 33 = 3(2+3) = 35

The next step is to calculate the value of 3 raised to the 5th power:

35 = 3 × 3 × 3 × 3 × 3 = 243

Therefore, the simplified expression is 243.

13 meters per second to miles per hour

Answers

Answer:

what I don't understand

Step-by-step explanation:

what exactly is the question

The following table lists the masses, in grams, of the eight planets and the dwarf planet Pluto. Use the data to choose the correct answers in the item below.

Uranus has a mass that is approximately___ times greater than Mercury's.
1. 3,000
2. 300
3. 30
4. 3

Uranus's mass = 8.61 x 10^28
Mercury's mass = 3.302 x 10^26


Answers

Answer:

Option 2) 300

Uranus has a mass that is approximately 300 times greater than Mercury's

Step-by-step explanation:

Let

x ----> Uranus's mass

y ---->  Mercury's mass

we have

[tex]x=8.61*10^28\ units[/tex]

[tex]y=3.302*10^26\ units[/tex]

Divide Uranus's mass by Mercury's mass

[tex]\frac{x}{y}[/tex]

substitute the values

[tex]\frac{8.61*10^28}{3.302*10^26}=260.75[/tex]

Round to the nearest hundred

therefore

Uranus has a mass that is approximately 300 times greater than Mercury's

Answer:

The answer is 300

Step-by-step explanation:

Estefani‘s house is at point E (3,-2) and Jasmin’s house is at point J (-5,3). Jasmin’s house is the Mid-point of Estefani’s house and Preston’s house. Give the y-coordinate of Preston’s house.

Answers

The formula for midpoint is

([tex]\frac{x_{1}+x_{2}}{2}[/tex], [tex]\frac{y_{1}+y_{2}}{2}[/tex])

Look at the image below to see the line segment of Estefani's (A), Jasmin's (M), and Preston's (B) houses. Keep in mind that the segment shown below is not accurate in regards to how the line segment formed by Estefani's, Jasmin's, and Preston's houses. It is simply there so you can picture the segment better.

In this case:

[tex]x_{1} =3\\x_{2} =unknown\\y_{1} =-2\\y_{2} =unknown[/tex]

^^^Plug in these number into the formula given above...

([tex]\frac{3+x}{2}[/tex], [tex]\frac{-2 + y}{2}[/tex])

To find what x is (a coordinate of Preston's house) you must take the x-value part of the midpoint equation ([tex]\frac{3+x}{2}[/tex]) and set it equal to the x-value of the midpoint (-5). Then you must solve for x:

[tex]\frac{3+x}{2}[/tex] = -5

3 + x = -5 * 2

3 + x = -10

x = -10 - 3

x = -13

To find what y is (a coordinate of Preston's house) you must take the  y-value part of the midpoint equation([tex]\frac{-2 + y}{2}[/tex]) and set it equal to the y-value of the midpoint (3). Then you must solve for y:

[tex]\frac{-2 + y}{2}[/tex] = 3

-2 + y = 3 * 2

-2 + y = 6

y = 6 + 2

y = 8

The coordinate of Preston's house is:

(-13, 8)

Hope this helped!

~Just a girl in love with Shawn Mendes

What is the solution for x in the equation?
16x-4+ 5x = -67
A
OB.
x = 1 1 1
r = 3
I = -3
se
-
cole

Answers

Answer:

x = -3

Step-by-step explanation:

[tex]16x-4+5x=-67\qquad\text{add 4 to both sides}\\\\16x+5x-4+4=-67+4\qquad\text{combine like terms}\\\\21x=-63\qquad\text{divide both sides by 21}\\\\\dfrac{21x}{21}=\dfrac{-63}{21}\\\\x=-3[/tex]

The solution for x in the equation is -3

What is the value of x ?

Given equation is, 16x-4+5x = -67

The given equation is a linear equation, so we will get only one value of x.

Solving the equation we get,

16x-4+5x = -67

⇒ 16x-4+4+5x = -67+4  (adding 4 in both sides)

⇒ 16x+5x = -63

⇒ 21x = -63

⇒ x = -63/21

⇒ x = -3

The required value of x = -3

Learn more about equation here :

https://brainly.com/question/27893282

#SPJ2

If a rectangle has a side with vertices located at (0, -8) and (10,-8), the length of the side is:

Answers

Answer:

10 units.

Step-by-step explanation:

That would be the difference in the x coordinates as the y coordinates are both -8.

So it's 10 - 0 = 10 units.

Whitney says that to add fractions with different denominators, you always have to multiply the denominators to find the common unit; for example:

1/4 + 1/6 = 6/24 + 4/24

Show Whitney how she could have chosen a denominator smaller than 24, and solve the problem.

Answers

Answer:

Step-by-step explanation:

You need to know what prime factorization is.  Basically you split up a number into it's prime  factors.  Or in other words what prime numbers multiply to get this number.  So say you have 36, you can split it into 2*2*3*3.  This is the methodology we use.

For the fractions given we want to look at the prime factorization of  the denominators.  so for 4 = 2*2 and 6 = 2*3.  What do we have to do to make them equal.  Well, if we multiply 2*2 by 3 and 2*3 by 2 what do we get?  2*2*3 and 2*3*2 which are equal.  So there's another way to get common denominators.  You also have to multiply the numerators by the same numbers you multiply the denominators by.  So in total you do this.

1/4 + 1/6

1/(2*2) + 1/(2*3)

(1*3)/(2*2*3) + (1*2)/(2*3*2)

3/12 + 2/12

5/12

Final answer:

To add fractions with different denominators, find the Least Common Denominator (LCD) rather than multiplying the denominators. For 1/4 and 1/6, the LCD is 12. The resulting addition is 1/4 + 1/6 = 3/12 + 2/12 = 5/12.

Explanation:

Whitney, to add fractions with different denominators more efficiently, we can find a common denominator that is smaller than directly multiplying the two denominators. This is also known as the Least Common Denominator (LCD).

To add 1/4 and 1/6, rather than multiplying the denominators to get 24, we can look for the least common multiple (LCM) of 4 and 6. The LCM of 4 and 6 is 12, because 12 is the smallest number that both 4 and 6 can divide evenly into.

Now we can write each fraction with a denominator of 12:

Multiply the first fraction (1/4) by 3/3 (which is equal to 1) to get 3/12Multiply the second fraction (1/6) by 2/2 (which is equal to 1) to get 2/12

Now the fractions have the same denominator:

1/4 + 1/6 = 3/12 + 2/12 = 5/12

By using the LCD, the fractions are now easy to add, and the added result of 5/12 is in simplified form.

10. A movie theater advertises that a family of two adults, one student, and one child between the ages of 3
and 8 can attend a movie for $15. An adult ticket costs as much as the combined cost of a student ticket
and a child ticket. You purchase 1 adult ticket, 4 student tickets, and 2 child tickets for $23.

Answers

Answer:

Students ticket = $4

Child ticket = $1

Adult ticket = $5

Step-by-step explanation:

Let the price of the student ticket be $x and the price of a child ticket be $y.  An adult ticket costs as much as the combined cost of a student ticket and a child ticket, so the price of one adult ticket is $(x+y).

1. A movie theater advertises that a family of two adults, one student, and one child between the ages of 3 and 8 can attend a movie for $15. Then

[tex]2(x+y)+x+y=15[/tex]

2. If you purchase 1 adult ticket, 4 student tickets, and 2 child tickets for $23, then

[tex](x+y)+4x+2y=23[/tex]

Now, solve the system of two equations:

[tex]\left\{\begin{array}{l}2(x+y)+x+y=15\\ \\(x+y)+4x+2y=23\end{array}\right.\Rightarrow \left\{\begin{array}{l}3(x+y)=15\\ \\5x+3y=23\end{array}\right.\\ \\\left\{\begin{array}{l}x+y=5\\ \\5x+3y=23\end{array}\right.\\ \\\left\{\begin{array}{l}y=5-x\\ \\5x+3(5-x)=23\end{array}\right.[/tex]

Solve the last equation

[tex]5x+15-3x=23\\ \\2x=23-15\\ \\2x=8\\ \\x=4\\ \\y=5-x=5-4=1[/tex]

Students ticket = $4

Child ticket = $1

Adult ticket = $5

Answer:

The ticket prices will as follows:

Students ticket = $4

Child ticket = $1

Adult ticket = $5

Step-by-step explanation:

Thinking process:

Let the price of the student ticket be $x and the price of a child ticket be $y.  An adult ticket costs as much as the combined cost of a student ticket and a child ticket, so the price of one adult ticket is $(x+y).

1. A movie theater advertises that a family of two adults, one student, and one child between the ages of 3 and 8 can attend a movie for $15. Then

2. If you purchase 1 adult ticket, 4 student tickets, and 2 child tickets for $23, then

Now, solve the system of two equations:

Solve the last equation

Students ticket = $4

Child ticket = $1

Adult ticket = $5

A verb form that functions as a noun is called a(n)

Answers

A verb form that functions as a noun is called a gerrund. :)

Answer:

It is called a gerund

Step-by-step explanation:

37 +(92÷3) what does it equal??​

Answers

The answer 67.6 and in fraction form it is 203/3

Phillip write an integer that is less than -2 and greater than -3.5 Which integer did he write?​

Answers

The answer would be -4

Answer:

The integer he wrote could be between -3.4 to -2.1, you could just say -3

Please answer this correctly

Answers

Gary swam the fewest laps.
Hope this helps!

Answer:

Gary

Step-by-step explanation:

Edward more than Gabby

So Edward>Gabby

But Edward swam fewer laps than Noelle

So Noelle>Edward>Gabby

Gabby swam more than Gary

So Noelle>Edward>Gabby>Gary

A pet shop has 562 gold fish. They use tanks that hold up to 18 fish in each. How many tanks will he need to buy to fit all the goldfish?​

Answers

Answer:

32 tanks

Step-by-step explanation:

since 562/18

is 31.222222...

You would have to round up because you the limit is 18 fish. You would need 32 tanks to fit all the fish

Answer:

32

Step-by-step explanation:

You have 562 gold fish and you need to have 18 in each tank, how many tanks do you need?

You first need to start with dividing the gold fish amount to the amount that can fit in each tank, (562 divided by 18 = 31.22222222222222). Then you multiply 18 by 31 and you get 558. This is obviously not as much goldfish as you had before, so to fix this you add another tank which makes 32 tanks, but you minus 562 from 558 and you have only 4 fish in the last tank.

Hope this helps! :)

What is -14=3x+x-2 can someone please help

Answers

Answer:

[tex]\large\boxed{x=-3}[/tex]

Step-by-step explanation:

[tex]-14=3x+x-2\qquad\text{add 2 to both sides}\\\\-14+2=3x+x-2+2\qquad\text{combine like terms}\\\\-12=4x\qquad\text{divide both sides by 4}\\\\\dfrac{-12}{4}=\dfrac{4x}{4}\\\\-3=x\to x=-3[/tex]

A store purchased an alarm clock and marked it up 175% from the original cost of $5.60. Then, wanting to make room for summer inventory, the store placed the clock on sale for 5% off. What was the price after the discount?

Answers

Answer:

$14.63

Step-by-step explanation:

175 × $5.60 ÷ 100 = 9.8

$5.60 + 9.8 = $15.40

5 × $15.40 ÷ 100 = $0.77

$15.40 - 0.77 = $14.63

720/91.2 = c/0.513 pleasse solve

Answers

720

91.2

=

c

0.513

Answer:

c

=

4.05

Answer:

c =  4.05  

Step-by-step explanation:

Given: 720/91.2 = c/0.513 .

To find: solve

Solution: We have given that  [tex]\frac{720}{91.2}[/tex]= [tex]\frac{c}{0.513}[/tex]    

 On cross multiplication

 [tex]\frac{720}{91.2}[/tex]= [tex]\frac{c}{0.513}[/tex]

91.2 * c = 0.513 * 720  

91.2 * c = 369.36

On dividing both side by 91.2

 c = [tex]\frac{369.36}{91.2}[/tex]

c =  4.05  

Therefore ,  c =  4.05  .

Is 0.6 repeated irrational?

Answers

0.6 is rational because it’s a terminating decimal, so no it’s not repeated irrational.

2y-3=3y

i need help

Answers

Answer:

y = -3

Step-by-step explanation:

First, subtract 2y from both sides, (3y - 2y) and that should give you the equation, -3 = y. And that's your final answer.

2y-3=3y

Move 2y to the other side

Sign changes from +2y to -2y

2y-2y-3=3y-2y

-3=y

Answer: y=-3

PLEASE HELP AND EXPLAIN!!!
I WILL MARK BRAINLIEST AND THANK YOU!!!

If DF = 42, find DE.

Answers

Answer:  The required length of DE is 29.

Step-by-step explanation:  We are given that DF = 42 in the figure shown.

We are to find the length of DE.

From the figure, we note that

[tex]DE=7x+1,~~EF=4x-3.[/tex]

According to the given information, we have

[tex]DF=42\\\\\Rightarrow DE+EF=42\\\\\Rightarrow 7x+1+4x-3=42\\\\\Rightarrow 11x=42+2\\\\\Rightarrow 11x=44\\\\\Rightarrow x=\dfrac{44}{11}\\\\\Rightarrow x=4.[/tex]

Therefore, we get

[tex]DE=7x+1=7\times4+1=28+1=29.[/tex]

Thus, the required length of DE is 29.

The length of DE using the measurements DF = 42, DE = 7x + 1 and EF = 4x - 3 is 29 unit.

given that,

the length of DF = 42

length of DE = 7x + 1

and, length of EF = 4x - 3

Now, from the figure

DE + EF = DF

Plugging the values back to the formula as

7x + 1 + 4x - 3= 42

Combine like terms

11x - 2 = 42

add 2 both side

11x = 42 + 2

11x = 44

Divide both side by 11

x= 44/11

x= 4

So, the length of DE is

= 7x + 1

= 28 + 1

= 29

Learn more about Equation here:

https://brainly.com/question/29657992

#SPJ3

Steps to solve the equation
-2(x-4)+8=2

Answers

Answer:

x=-1

Step-by-step explanation:

-2(x+4)+8=2

1) Use the Distributive property:

-2x-8+8=2

2) Combine alike terms:

-2x=2

3) Divide both sides by -2:

x=-1

Which statement is true?


A. Figure A is congruent to figure B.
B. Figure B is congruent to figure C.
C. Figure C is congruent to figure D.
D. Figure D is congruent to figure A.

Answers

Answer:

B. Figure B is congruent to figure C.

Step-by-step explanation:

Figure D is dilated about a quarter of the other two, and Figure A is out of the question because it is dilated about twice their size, so with that being said, you have your answer.

I am joyous to assist you anytime.

Answer:

b

Step-by-step explanation:

Other Questions
What are the 3 ways that an object can change its velocity? What two principles did Providence guarantee that Massachusetts Bay did not? A hot-air balloon is descending at a rate of 2.1 m/s when a passenger drops a camera. If the camera is 42 m above the ground when it is dropped, how long does it take for the camera to reach the ground?Express your answer using two significant figures.If the camera is 42 m above the ground when it is dropped, what is its velocity just before it lands? Let upward be the positive direction for this problem.Express your answer using two significant figures. The letter J is formed by sticking up the smallest finger and folding all the other fingers into the palm of your hand with the thumb folded over them. Which sentence corrects the error in pronoun-antecedentagreement?New team members should pick up his or her jerseyfrom the activities office.If a team member is unable to attend a practice, he orshe must provide a doctor's note.If students wish to compete, he or she must maintain agrade point average above 2.5.Students are not permitted to attend away gameswithout the consent of her parents or guardians.1SEDEELLEill Why would an image of a hero change over time? The square matrix A is called orthogonal providedthatAT=A-1. Show that the determinant of suchamatrix must be either +1 or -1. For jet aircraft engine, what is not used for the system? A- nozzle B- Turbine C- compressor D- none After Maya told her friend the 5-digit password to a protected website, her friend was able to remember it only long enough to type it into the password box. In this instance, the password was clearly stored in her friend's ________ memory.proceduralshort-termiconicimplicit write an integer for the situation "An average temperature of 7 degrees below normal A circle with radius 9 has a sector with a central angle of 120. What is the area of the sector. Provide step by step explanation please A projectile is launched from ground level with an initial velocity of v 0 feet per second. Neglecting air resistance, its height in feet t seconds after launch is given by s equals negative 16 t squared plus v 0 t. Find the time(s) that the projectile will (a) reach a height of 192 ft and (b) return to the ground when v 0equals128 feet per second. All fungi area. symbiotic.b. heterotrophic.c. flagellated.d. decomposers Why is reaching activation energy necessary? Reactants require a minimum amount of energy to start to break a chemical bond. Products require a minimum amount of energy to start to break a chemical bond. Reactants must have the maximum amount of energy to start to break a chemical bond. Products must have the maximum amount of energy to start to break a chemical bond. The mB is two more than three times the measure of C. If B and C are complementary angles, find the measures of both angles. U.S History What are 10 oral history interview questions that you can think of? What innovations from the Hellenistic Age are still used in our world today? What is the value of 5 in 5,476,807,139 Which of the following terms best represents a situation in which a government does not attempt to influence through quotas or duties what its citizens can buy from another country, or what they can produce and sell to another country? A. Free trade B. Positive-sum game C. Socialism D. Absolute advantage E. Zero-sum game Question 14Antacids work because they _____ excess stomach acid.A)neutralizeB)containC)acidifyD)titrate